Sei sulla pagina 1di 9

MA2108S Tutorial 4 Answer Sheet

Qian Yujie, Shi Yangyang, Xu Jingwei


21 February 2011

Section2.4
9. Perform the computations in (a) and (b) of the preceding exercise for the function
h : X × Y → R defined by (
0 if x < y,
h(x, y) :=
1 if x ≥ y.

Proof. (a) Note that h(y, y) = 1, hence f (x) := sup{h(x, y) : y ∈ Y } = 1 for all x ∈ X.
Thus inf{f (x) : x ∈ X} = 1.
1+x 1+x
(b) For each x ∈ X, we take y = , then we have h(x, ) = 0,then g(y) =
2 2
inf{h(x, y) : x ∈ X} = 0 for all y ∈ Y . Thus sup{g(y) : y ∈ Y } = 0.

10. Let X and Y be nonempty sets and let h : X × Y → R have bounded range in R. Let
f : X → R and g : Y → R be defined by

f (x) := sup{h(x, y) : y ∈ Y }, g(x) := inf{h(x, y) : x ∈ X}.

Prove that

sup{g(y) : y ∈ Y } ≤ inf{f (x) : x ∈ X}.

Proof. Since f (x0 ) ≥ h(x0 , y)∀ y ∈ Y , and g(y0 ) ≤ h(x, y0 ) ∀ x ∈ X. Take any x0 ∈ X,
then f (x0 ) ≥ h(x0 , y) ≥ g(y)∀ y ∈ Y , namely f (x) ≥ g(y) ∀ x ∈ X ∀ y ∈ Y .
Hence f (x) is an upper bound of {g(y) : y ∈ Y }, sup{g(y) : y ∈ Y } ≤ f (x) ∀ x ∈ X,
Therefore sup{g(y) : y ∈ Y } is a lower bound of {f (x) : x ∈ X}.
Hence sup{g(y) : y ∈ Y } ≤ inf{f (x) : x ∈ X}.

11. Let X and Y be nonempty sets and let h : X × Y → R have bounded range in R.
Let F : X → R and G : Y → R defined by

F (x) := sup{h(x, y) : y ∈ Y }, G(y) := sup{h(x, y) : x ∈ X}.

1
Establish the Principle of the Iterated Suprema

sup h(x, y) = sup sup h(x, y) = sup sup h(x, y).


x,y x y y x

Proof. ∀ x0 ∈ X, y ∈ Y, h(x0 , y) ≤ F (x0 ). Hence sup h(x, y) ≤ sup{F (x) : ∀ x ∈ X}.


x,y
On the other hand, sup h(x, y) ≥ h(x0 , y) ∀x0 ∈ X, y ∈ Y , hence sup h(x, y) ≥ sup{F (x) :
x,y x,y
x ∈ X}. Hence sup h(x, y) = sup{F (x) : ∀ x ∈ X} = sup sup h(x, y).
x,y x y
By symmetry, sup h(x, y) = sup sup h(x, y).
x,y y x
Hence sup h(x, y) = sup sup h(x, y) = sup sup h(x, y).
x,y x y y x

12. Given any x ∈ R, show that there exists unique n ∈ Z such that n − 1 ≤ x < n.

Proof. (1) If x = 0, then let n := 1, n − 1 = 0 ≤ x < 1 = n.

(2) If x > 0, Let S := {N : n > x}. By the Archimedean Property, ∃ n ∈ N, such


that n > x, S is nonempty. By the well ordering property, a smallest element n0 can
be picked in S. Since n0 − 1 < n0 , therefore n0 − 1 ∈
/ S. Namely n0 − 1 ≤ x. Hence
n0 − 1 ≤ x < n0 .

(3) If x < 0, then −x > 0. By the Archimedean Property, ∃ m ∈ N with m > −x.
Therefore m + x > 0. By the result of part (2), ∃ n0 ∈ N n0 − 1 ≤ m + x < n0 . Hence
(n0 − m) − 1 ≤ x < (n0 − m) with (n − m) ∈ Z. Take n := n0 − m, then n − 1 ≤ x < n.

Uniqueness: suppose to the contrary, for any x ∈ R, ∃ m, n ∈ Z and m 6= n, such


that n − 1 ≤ x < n and m − 1 ≤ x < m. Without loss of generality, assume m > n,
because m, n ∈ Z, therefore m ≥ n + 1. Then, m − 1 ≥ n > x, which contradicts with
the fact that m − 1 ≤ x, therefore m = n.

Hence given any x ∈ R, there is a unique n ∈ Z such that n − 1 ≤ x < n.

1
13. If y > 0, show that there exists n ∈ N such that < y.
2n
1
Proof. Since y > 0, by The Archimedean Property, ∃n ∈ N, such that < y. Since
µ ¶ n
n 1 1
2n = (1 + 1)n ≥ 1 + > n for n ∈ N , n < < y.
1 2 n
1
Hence if y > 0, there exists n ∈ N such that n < y.
2

2
18. If u > 0 is any real number and x < y, show that there exists a rational number r
such that x < ru < y. (Hence the set {ru : r ∈ Q is dense in R.)
x y y x
Proof. Since u > 0, and x < y, therefore < . Since − > 0, by the Archimedean
u u u u
1 y x x·n y·n
Property, ∃ n ∈ N, such that < − . Hence 1 + < . Apply Corollary
n u u u u
x·n
2.4.6 (or result of Exercise 2.4.12), ∃ m ∈ N, such that m − 1 ≤ < m. Therefore,
u
x·n y·n x·n y·n m
m≤ +1 < , whence <m< . Thus the rational number r :=
u u u u n
satisfies x < ru < y.

ExtraA Let A and B be nonempty subsets of R. Let

A + B = {a + b : a ∈ A, b ∈ B}. and AB = {ab : a ∈ A, b ∈ B}.

Show that inf(A+B) = inf A+inf B. If A and B are both subsets of positive real number,
show also that inf(AB) = inf A inf B.
Proof. (1) ∀a ∈ A, ∀b ∈ B, inf A + inf B ≤ a + b. Thus inf A + inf B is a lower bound of
A + B.
For any ² > 0, ∃ a 2² ∈ A, such that a 2² < inf A+ 2² ; and ∃ b 2² ∈ B, such that b 2² < inf B + 2² .
Therefore, for any ² > 0, ∃ a 2² ∈ A, ∃ b 2² ∈ B, such that a 2² + b 2² < inf A + inf B + ². By
Lemma 2.3.4 inf A + inf B = inf(A + B).

(2)∀ a ∈ A a ≥ inf A ≥ 0. ∀ b ∈ B, b ≥ inf B ≥ 0. Hence ∀ a ∈ A, ∀b ∈ B, inf A·inf B ≤


a · b. Hence inf A · inf B is a lower bound of AB. Let u be a lower bound of AB.
u u
Then ∀ a ∈ A, ∀b ∈ B, ab ≥ u. a, b > 0, thereforea ≥ , is a lowerbound of A.
b b
u
Henceinf A ≥ which implies u ≤ inf A · b. Since inf A ≥ 0,
b
u u
Case 1: if inf A > 0 then ∀ b ∈ B b ≥ . Therefore inf B ≥ then inf A·inf B ≥ u;
inf A inf A
Case 2: if inf A = 0, y > 0, therefore u = 0, thus ∴ inf B · inf A = 0 = u. u is any
lower bound of AB, hence inf A · inf B = inf(AB)

Hence inf A · inf B = inf(AB).

3
Section 2.5
Question 1
If I := [a, b] and I 0 := [a0 , b0 ] are closed intervals in R, show that I ⊆ I 0 if and only if
a0 ≤ a and b ≤ b0 , which leads to contradiction.

Proof. If ∀a, b ∈ I ⇒ a, b ∈ I 0 , then a0 ≤ a, b ≤ b0


Conversely, if a ≤ a, b ≤ b0 , then a0 ≤ x ≤ b0 whenever a ≤≤ x ≤ b. Thus I ⊆ I 0 .

Question 2
If S ⊆ R is nonempty, show that S is bounded if and only if there exists a closed bounded
interval I such that S ⊆ I.
Proof. (1) Claim: The set S is bounded if there exists a closed bounded interval I such
that S ⊆ I.
Take I = [a, b], then for any x ∈ S, a ≤ x ≤ b.
Hence for any x ∈ S, a ≤ x ⇒ S is bounded below.
For any x ∈ S, x ≤ b ⇒ S is bounded above.
Hence S is bounded.
The set S is bounded if there exists a closed bounded interval I such that S ⊆ I.
(2) Claim: The set S is bounded only if there exists a closed bounded interval I such that
S ⊆ I.
As S is bounded above, there exists a number u ∈ R such that s ≤ u for all s ∈ S.
Also, S is bounded below, there exists a number w ∈ R such that w ≤ s for all s ∈ S.
Hence w ≤ s ≤ u.
Hence there exists closed bounded interval I = [w, u] such that S ⊆ I.
Hence S is bounded only if there exists a closed bounded interval I such that S ⊆ I.
Hence S is bounded if and only if there exists a closed bounded interval I such that
S ⊆ I.

Question 3
If S ⊆ R is a nonempty bounded set, and Is := [inf S, supS], show that S ⊆ Is . Moreover,
if J is any closed bounded interval containing S, show that Is ⊆ J.
Proof. (1) Claim: The set S ⊆ Is .
Take any s ∈ S, then inf S ≤ s ≤ sup S ⇒ s ∈ Is . Hence S ⊆ Is .

(2) Claim: The set Is ⊆ J if S ⊂ J.


Suppose J = [a, b]. Then for any s ∈ S, since S ⊂ J, we have a ≤ s ≤ b. Hence a is a
lower bound of S and b is an upper bound of S. Therefore a ≤ inf S.
For any s ∈ S,s ∈ J, then b ≥ s. Therefore b is a upper bound of S. Hence sup S ≤ b ≤
a ≤ b.
Hence Is ⊆ J by Question 1. Hence if J is any closed bounded interval containing S,
Is ⊆ J.

4
Question 7

\
Let In := [0, 1/n] for n ∈ N. Prove that In = {0}.
n=1

\
Proof. Clearly 0 ∈ In for any n ∈ N . Hence 0 ∈ In .
n=1

\
Claim: The set {0} = In .
n=1

\ ∞
\ ∞
\
Suppose {0} 6= In . Since 0 ∈ In , there exists a x 6= 0, such that x ∈ In .Hence
n=1 n=1 n=1
x ∈ In for all n ∈ N . As In = [0, 1/n], x > 0. Hence by the Archimedean Property,

\
there exists a K ∈ N such that 1/K < x. Hence x ∈ / IK . Hence x ∈
/ In . This is a
n=1
contradiction!

\
Hence In = {0}.
n=1

Question 8

\
Let Jn := (0, 1/n) for n ∈ N. Prove that Jn = ∅.
n=1

\
Proof. Suppose there exists a x, such that x ∈ Jn . Hence x ∈ Jn for all n ∈ N . As
n=1
Jn = (0, 1/n), we have x > 0. Hence by the Archimedean Property, there exists a K ∈ N

\
such that 1/K < x. Hence x ∈ / JK . Hence x ∈
/ Jn . This is a contradiction!
n=1

\
Hence Jn = ∅.
n=1

Question 9

\
Let Kn := (n, ∞) for n ∈ N. Prove that Kn = ∅.
n=1

5

\
Proof. Suppose there exists a x, such that x ∈ Kn . Hence x ∈ Kn for all n ∈ N . Also,
n=1
Kn = (n, ∞), we have x > 0. Hence by the Archimedean Property, there exists a K ∈ N
\∞
such that x < K. Hence x ∈
/ KK . Hence x ∈/ Kn . This is a contradiction!
T n=1
Hence ∞ n=1 Kn = ∅.

Question 10
With the notation in the proofs of Theorems 2.5.2 and 2.5.3, show that we have η ∈

\
T∞
n=1 In . Also show that [ξ, η] = In .
n=1

Proof. Recall that η = inf{bn : n ∈ N} and the fact that for each n ∈ N, an is a lower
bound of the set {bn : n ∈ N}, it is clear that η ≥ an for each n. Thus η ≥ ξ = sup{an :
n ∈ N}.
Next, it is clear that if x ∈ [ξ, η], then an ≤ ξ ≤ Tx ≤ η ≤ bn for any n. Thus
x ∈ [an , bn ] = In for each
T n. It is then clear that [ξ, η] ⊂ n∈N In .
Conversely, if x ∈ n∈N In , then an ≤ x ≤ bn for all n and hence x is an upper
bound of T {an : n ∈ N} and a lower bound of {bn : n ∈ N}. Thus ξ ≤ x ≤ η and hence
[ξ, η] ⊂ n∈N In . T
Thus [ξ, η] = n∈N In .

Question 14
a1 a2 an b1 b2 bm
14.Show that if ak , bk ∈ {0, 1, . . . , 9} and if + 2 +· · ·+ n = + 2 +· · ·+ m 6= 0,
10 10 10 10 10 10
then ak = bk for k = 1, · · · , n.
ak bk
Proof. Suppose not. Then there is a least number i such that k = k when k < i and
10 10
ai bi
6= i .
10i 10
ai bi
Without loss of generality, we may assume i > i . Since ai , bi ∈ {0, 1, . . . , 9}, we
10 10
ai bi 1
have ai ≥ bi + 1. Hence i ≥ i + i . Note that
10 10 10
bi+1 bi+2 bm 9 9 9
i+1
+ i+2 + · · · + m ≤ i+1 + i+2 + · · · + m
10 10 10 10 10 10
9 1 m−i
i+1 [1 − ( 10 ) ] 1 1 1
= 10 1 = i [1 − m−i ] < i .
1 − 10 10 10 10

6
Then we have
ai bi 1 bi bi+1 bi+2 bm
i
≥ i + i > i + i+1 + i+2 + · · · + m .
10 10 10 10 10 10 10
Hence
a1 a2 an a1 a2 ai b1 b2 bn
+ 2 + ··· + n ≥ + 2 + ··· + i > + 2 + ··· + n.
10 10 10 10 10 10 10 10 10
This is a contradiction.
Hence ak = bk for all k.

Question 16
1 2
Express and as periodic decimals.
7 19
1
Solution: By computation, we have = 0.142857 · · · 142857 · · ·
7
2
and = 0.105 263 157 894 736 842 · · · 105 263 157 894 736 842 · · · .
19

Question 17
What rationals are represented by the periodic decimals 1.25137 · · · 137 · · · and 35.14653 · · · 653 · · · ?

Solution: Take x = 1. 25137 · · · 137 · · · . Then we have (100000 − 100)x = 125012. Hence
x = 31253/24975. Hence 1. 25137 · · · 137 · · · = 31253/24975.
Take y = 35. 14653 · · · 653 · · · . Then we have (100000 − 100)x = 3511139. Hence
x = 3511139/99900. Hence 35. 14653 · · · 653 · · · = 3511139/99900.

Extra Question (C):


Let z > 0. Show that for any n ∈ N, there exists x > 0 such that xn = z.
Proof. Case 1: z > 1
Let S = {s ∈ R : s ≥ 0, sn < z}.
Since 1 ∈ S, then S is not an empty set. Also, S is bounded above by z, as z n > z and
sn < z n ⇔ s < z (recall that s, z ≥ 0). Therefore, the Completeness Property implies
that the set S has a supremum in R. Let x = supS. Note that x > 1.
First assume that xn < z.
(x+δ)n −xn = (x+δ−x)((x+δ)n−1 +(x+δ)n−2 x+· · ·+xn−1 ) < δ(x+δ)n−1 n < δ(x+1)n−1 n,
for n ≥ 2, n ∈ N and 0 < δ < 1.
Since z − xn > 0, by the Archimedean Property, there exists N ∈ N such that z − xn > N1 .
1 1
Let δ = n−1
. Then (x + δ)n − xn < δ(x + 1)n−1 n = < z − xn .
N (x + 1) n N

7
Hence (x + δ)n < z. It is a contradiction to the fact x := supS. Hence xn ≮ z.
Next assume that xn > z.
Then xn − (x − δ)n = (x − x + δ)(xn−1 + xn−2 (x − δ) + · · · + (x − δ)n−1 ) < δxn−1 n, for
n ≥ 2, n ∈ N and 0 < δ < 1.
1
Since xn −z > 0, by the Archimedean Property, there exists N ∈ N such that xn −z > .
N
1 n n n−1 1 n
Let δ = . Then x − (x − δ) < δx n = < x − z.
N xn−1 n N
Hence (x − δ)n > z. It is a contradiction to the fact x := supS. Hence xn ≯ z.
Combining these two cases, we conclude that xn = z.

Case 2: z = 1
Clearly, if x = 1, then xn = 1.
Case 3: 0 < z < 1
Then 1/z > 1. By Case (1), there exists x > 1 such that xn = 1/z. Observe that
(1/x)n = z.

Section3.1

Question: 5(a)
n
Use the definition of the limit of a sequence to establish the limit of lim( 2 ) = 0.
n +1
¯ ¯
¯ n ¯
¯
Proof. Observe that ¯ 2 ¯ < n = 1.
n + 1 ¯ n2 n
1
Given any ² > 0, by the Archimedean Property, there exists k ∈ N, such that < ².
k
1 1
Therefore if n ≥ k, then ≤ < ².
n k
n n
Hence, | 2 | < ², for all n ≥ k. Hence lim( 2 ) = 0.
n +1 n +1

Question: 5(d)
n2 − 1 1
Use the definition of limit of a sequence to establish the limit of lim( 2
)= .
2n + 3 2
n2 − 1 1 2n2 − 2 − 2n2 − 3 5 5 5 5
Proof. Observe that | 2
− | = | 2
|=| 2 |< 2 < 2 ≤ .
2n + 3 2 4n + 6 4n + 6 4n n n
1 ²
Given ² > 0, by the Archimedean Property, there exists k ∈ N, such that < .
n 5
5 5
Therefore take n ≥ k, then ≤ < ².
n k
n2 − 1 1 5
Hence, | 2 − | < < ², for all n ≥ k.
2n µ+ 3 2 ¶ n
n2 − 1 1
Hence, lim 2
= .
2n + 3 2

8
Question: 6(c)
1
Show that lim( √ ) = 0.
n+7
√ √
n n 1
Proof. Observe that | |< =√ .
n+7 n n
1
Given any ² > 0, by the Archimedean Property, there exists k ∈ N, such that k
< ²2 .
1 1 1
Therefore if n ≥ k, the ≤ ≤ ²2 , thus √ < ².
√ n k n
n
Hence, | | < ², for all n ≥ k.
n+1

Question: 6(d)
(−1)n
Show that lim( ) = 0.
n2 + 1
(−1)n n n n 1
Proof. Observe that | 2
|≤| 2 | < | 2| = .
n +1 n +1 n n
1 ²
Given any ² > 0, by the Archimedean Property, there exists k ∈ N, such that < .
k 5
1 1
Therefore if n ≥ k, then ≤ < ².
n k
(−1)n n
Hence, | 2 | < ², for all n ≥ k.
n +1

Potrebbero piacerti anche